Exercise: is f(x,y) continuous and differentiable?

In summary: I don't understand what you're trying to say. :(Sorry, I didn't read this post properly the first time (jet lag). You claim to have shown f < some other function, g say, then because g tends to...well, I don't understand what you're trying to say. :(
  • #1
Felafel
171
0

Homework Statement



could you please check if this exercise is correct?
thank you very much :)

##f(x,y)=\frac{ |x|^θ y}{x^2+y^4}## if ##x \neq 0##
##f(x,y)=0## if ##x=0##
where ##θ > 0## is a constant
study continuity and differentiabilty of this function

The Attempt at a Solution


Being the quotient of continuous functions, it is always continuos except, maybe, in the point (0,0)
I normally use the sequential continuity to tell is a function of this kind is continuous. However, I couldn't find a couple of suitable successions for this case, so I decided to proceed as follows:
## \left| \frac{|x|^θ y}{x^2+y^4} - f (0,0) \right| ≤ \left| |x|^θ y \right|##
that goes to 0 if x,y go to 0. So the function is continuous in (0,0).
Then it could be differentiable. I look for the partials:

##\frac{\partial f}{\partial x}\lim_{t \to 0} \frac{f(x_0+t)-f(0,0)}{t}=0##
and
##\frac{\partial f}{\partial y}=0## as well. if they are continuous in 0,0 the function is differentiable for the theorem of total differential. but the partials ar every "complicated", so i prefer to apply the definition of differential:

##\lim_{(x,y) \to (0,0)} \frac{f(x,y)-f(0,0)-(∇f(0,0),(x,y)}{\sqrt{x^2+y^2}}##
=## \left| \frac{|x|^θ y}{(x^2+y^4) \sqrt{x^2+y^2}}-0 \right| ## but i see it is my original function multiplied ##\frac{1}{\sqrt{x^2+y^2}}##. this last function goes to infinity for x,y going to 0,0, so the limit doesn't exist and the function is not differentiable.
 
Physics news on Phys.org
  • #2
Felafel said:
## \left| \frac{|x|^θ y}{x^2+y^4} - f (0,0) \right| ≤ \left| |x|^θ y \right|##
I don't follow the logic of that. You're taking f(0,0) to be 0, I presume.
 
  • #3
Hint:
Introduce polar coordinates, and see for which values of your constant "theta" the function is:
a) Continuous
b) And furthermore, differentiable.
 
  • #4
If a function is differentiable, then it is continuous. Since you are asked whether or not the function is differentiable and continuous, I think I would be inclined to look at "differentiable" first. If the answer is "yes, it is differentiable for all (x, y)", you get "continuous" automatically!

Of course, "differentiable" for functions of two variables is more complicated than just saying the partial derivatives exist.
 
  • #5
Halls:
Sure enough.
But, in this case, you have parameter values for which the function is continuous, but not differentiable, and that range ought to be included when saying when the function is continuous.
 
  • #6
haruspex said:
I don't follow the logic of that. You're taking f(0,0) to be 0, I presume.

yes, exactly, it's part of the hypothesis given
 
  • #7
arildno said:
Hint:
Introduce polar coordinates, and see for which values of your constant "theta" the function is:
a) Continuous
b) And furthermore, differentiable.

we haven't studied polar coordinates during the course (yet?). isn't there another way?
 
  • #8
Felafel said:
yes, exactly, it's part of the hypothesis given
Sure, but how do you obtain the inequality I quoted? It seems to require the denominator, x2+y4, to be ≥ 1.
 
  • #9
haruspex said:
Sure, but how do you obtain the inequality I quoted? It seems to require the denominator, x2+y4, to be ≥ 1.
yes, i just figured out later that it might be false.
then sould i try to do this:
##(x-y^2)^2 \geq 0 \to x^2+y^4 \geq xy^2##
so
## \left| \frac{|x|^θy}{y^4+x^2} \right| \leq \left| \frac{|x|^θ}{xy}\right|##
if
##x=\frac{1}{n}## i get ##y=|\frac{1}{n}|^θ \cdot n## and for n to infinity the f(x,y) goes to 0 only if θ>2. so for θ<2 it is not continuous
 
  • #10
Felafel said:
yes, i just figured out later that it might be false.
then sould i try to do this:
##(x-y^2)^2 \geq 0 \to x^2+y^4 \geq xy^2##
so
## \left| \frac{|x|^θy}{y^4+x^2} \right| \leq \left| \frac{|x|^θ}{xy}\right|##
if
##x=\frac{1}{n}## i get ##y=|\frac{1}{n}|^θ \cdot n## and for n to infinity the f(x,y) goes to 0 only if θ>2. so for θ<2 it is not continuous
Yes, that's better, but what about θ=2? Next, differentiability.
 
  • #11
haruspex said:
Yes, that's better, but what about θ=2? Next, differentiability.
it's not contnuous in (0,0) for θ=2, because it's homogeneus of degree 0.
I know it can't be differentiable when it isn't continuous, so , using the same proceeding as that i posted at the beginning, i'll say that it is differentiable for every θ>2
 
  • #12
Felafel said:
it's not contnuous in (0,0) for θ=2, because it's homogeneus of degree 0.
You are right that it is not continuous for θ=2, but I don't understand your reasoning. Wouldn't f(x,y)=0 be homogeneous of degree 0?
I know it can't be differentiable when it isn't continuous, so , using the same proceeding as that i posted at the beginning, i'll say that it is differentiable for every θ>2
In your OP you concluded it was nowhere differentiable (which was wrong). If you now believe it is differentiable for every θ>2 pls post your proof.
 
  • #13
Felafel said:
yes, i just figured out later that it might be false.
then sould i try to do this:
##(x-y^2)^2 \geq 0 \to x^2+y^4 \geq xy^2##
so
## \left| \frac{|x|^θy}{y^4+x^2} \right| \leq \left| \frac{|x|^θ}{xy}\right|##
if
##x=\frac{1}{n}## i get ##y=|\frac{1}{n}|^θ \cdot n## and for n to infinity the f(x,y) goes to 0 only if θ>2. so for θ<2 it is not continuous
Sorry, I didn't read this post properly the first time (jet lag). You claim to have shown f < some other function, g say, then because g tends to infinity at O for a certain theta you conclude f does too. That's illogical. You would need to show f >= g for that. And I should not have agreed that f is discontinuous at O for all theta < 2.
Try considering lines of approach to the origin, like y = kx for a constant k.
 
  • #14
haruspex said:
Sorry, I didn't read this post properly the first time (jet lag). You claim to have shown f < some other function, g say, then because g tends to infinity at O for a certain theta you conclude f does too. That's illogical. You would need to show f >= g for that. And I should not have agreed that f is discontinuous at O for all theta < 2.
Try considering lines of approach to the origin, like y = kx for a constant k.

no worries :)
unfortunately i have never heard about the method you mentioned, and our teacher wants us to solve it the way he taught us, which means by increasing/reducing inequalities and using a few theorems (lagrange, theorem of the total differential...)
but i thought that I could show the greater inequality converges to 0, so that the first one also does:

## \left|\frac{|x|^θy}{x^2+y^4} \right| \leq \left|frac{|x|^θy}{xy^2} \right| = \left|frac{|x|^θ}{xy} \right|##
now if ##x=\frac{1}{n}## i get ##y=\frac{1}{n^{θ-1}}##
so the function becomes: ##\frac{\frac{1}{n}}{\frac{1}{n^{θ-1}}} \to ##n^{θ-2}## that goes to zero if ##θ-2<|1|## so ##θ-2>-1## and ##θ-2>1## that gives
##1<θ<3##. for these values it is continuous.
I look for differentiabilty just like in the OP and i get the function is not differentiable in (0,0)
 
  • #15
Fixing up your LaTex:
Felafel said:
## \left| \frac{|x|^θy}{x^2+y^4} \right| \leq \left| \frac{|x|^θy}{xy^2} \right| = \left| \frac{|x|^θ}{xy} \right|##
now if ##x=\frac{1}{n}## i get ##y=\frac{1}{n^{θ-1}}##
I don't understand the logic of that. How does it give you a value for y? To deduce that you would need that the RHS = 1.
The RHS is an upper bound for the function, so you cannot us it for showing discontinuity. At est, you can use it to show continuity by showing the RHS tends to 0 as the origin is approached. But that is not the case, since it is unbounded along y=0, which the original function was not.
Can you describe in more detail the methods you have been taught for testing continuity of functions of two variables?
The line-of-approach method is a quick and easy way to get an idea of what is going on. You consider approaching the point of interest along some curve - in this case a line like y = kx. That gets it down to a function of one variable. If this produces the right limit irrespective of k then you've a good chance it is continuous (but you at least also need to check the case of approaching along the y axis, which the values of k do not cover). If there is any value of k which produces the 'wrong' limit then it definitely is not continuous.
Another useful way is converting to some sort of polar form. In this case, because you have y^4, I would suggest ##r^2 = x^2 + y^4##, so ## x = r \cos(\theta)##, ## y^2 = r \sin(\theta) ##. (There's clearly a problem with this in the 3rd and fourth quadrants - not sure how to handle that.) You'd then need to show two things: f tends to 0 as r tends to 0, regardless of theta; as |(x, y)| tends to 0 r tends to 0.
I look for differentiabilty just like in the OP and i get the function is not differentiable in (0,0)
The method in the OP is wrong. |(x,y)| tends to 0 does not imply f(x,y)/|(x,y)| tends to infinity. f might tend to zero even faster. I assure you there are values of theta for which f is differentiable at O.
 
  • #16
haruspex said:
Fixing up your LaTex:

Can you describe in more detail the methods you have been taught for testing continuity of functions of two variables?
yes, I've been taught these theorems:
lagrange (mean value)
theorem of the total differential
clauiraut/schwarz's theorem
but they don't seem to be very helpful for the continuity problem, for which i should use just the manipulation of inequalities with the help of young's inequality, for instance, and using the sequential continuity, maybe by subdiving a limit in two limits one of those goes to zero and the other to a real number
i'll give you an example: i want to study the continuity in (0,0) of
f(x,y)=##\frac{x^3 y}{x^4+y^2}## if ##x,y\neq(0,0)##
##f(x,y)=0## if ##(x,y)=(0,0)##
##0 \leq \lim_{(x,y) \to (0,0)}\frac{|x|^3 |y|}{x^4+y^2}\leq \lim_{(x,y) \to (0,0)} |x|\frac{|x|^4+ |y|^2}{2(x^4+y^2)}## by cauchy's inequality
and, finally:
##\lim_{(x,y) \to (0,0)}\frac{|x|}{2}=0## which means the function is continuous
 
  • #17
Felafel said:
yes, I've been taught these theorems:
lagrange (mean value)
theorem of the total differential
clauiraut/schwarz's theorem
but they don't seem to be very helpful for the continuity problem, for which i should use just the manipulation of inequalities with the help of young's inequality, for instance, and using the sequential continuity, maybe by subdiving a limit in two limits one of those goes to zero and the other to a real number
i'll give you an example: i want to study the continuity in (0,0) of
f(x,y)=##\frac{x^3 y}{x^4+y^2}## if ##x,y\neq(0,0)##
##f(x,y)=0## if ##(x,y)=(0,0)##
##0 \leq \lim_{(x,y) \to (0,0)}\frac{|x|^3 |y|}{x^4+y^2}\leq \lim_{(x,y) \to (0,0)} |x|\frac{|x|^4+ |y|^2}{2(x^4+y^2)}## by cauchy's inequality
and, finally:
##\lim_{(x,y) \to (0,0)}\frac{|x|}{2}=0## which means the function is continuous
Well I can't see how to use those either. Besides, fishing around for a useful inequality is a rather haphazard approach. In the present case, what tends to happen is that you show f is bounded above by some function g, but g is unbounded near the origin, so it proves nothing.
My method, encapsulating the denominator in a single variable which, as it tends to zero, will force x and y to zero, is more deterministic.
The 3rd and 4th quadrants aren't a problem after all. I'm defining ##r = \sqrt{x^2+y^4}## and ##\phi = \arctan(x/y^2)##. You can show f is bounded above by r to some power. As long as that power > 0 you have continuity. Next, you can show that, when that power = 0, f is not continuous, completing the proof.
If you are determined to find a proof along the lines you have been taught, you could use my method to find what the critical value of theta is and see if that gives you any clues.
 
Last edited:
  • #18
yes, I really can't use polar coordinates. But using y=kx in ##\frac{|x|^{\Theta}}{xy}## i see that I have to have ##\Theta>2##, otherwise the denominator goes to 0 faster then the numerator. right?
 
  • #19
Felafel said:
yes, I really can't use polar coordinates. But using y=kx in ##\frac{|x|^{\Theta}}{xy}## i see that I have to have ##\Theta>2##, otherwise the denominator goes to 0 faster then the numerator. right?
##\frac{|x|^{\Theta}}{xy}## is not the function you are given - it is only a bound which you have found for the function. Unfortunately it is of no use. If you come in on the x-axis you will have y = 0, so your upper bound is infinite, and tells you nothing at all about f.
Why can't you use polar coordinates? It's just a substitution into different variables.
 

Related to Exercise: is f(x,y) continuous and differentiable?

1. What is the definition of continuity?

The definition of continuity for a function f(x,y) is that it is continuous at a point (a,b) if the limit of the function as (x,y) approaches (a,b) is equal to the value of the function at (a,b).

2. How can I determine if a function is continuous?

A function f(x,y) is continuous if it satisfies the following conditions:

  • It is defined at the point (a,b).
  • The limit of the function as (x,y) approaches (a,b) exists.
  • The value of the limit is equal to the value of the function at (a,b).

3. What does it mean for a function to be differentiable?

A function f(x,y) is differentiable at a point (a,b) if the limit of the difference quotient as (h,k) approaches (0,0) exists. This means that the function has a well-defined slope at that point.

4. How do I determine if a function is differentiable?

A function f(x,y) is differentiable if it satisfies the following conditions:

  • It is defined at the point (a,b).
  • The limit of the difference quotient as (h,k) approaches (0,0) exists.
  • The partial derivatives of the function exist at (a,b).

5. Can a function be continuous but not differentiable?

Yes, a function can be continuous but not differentiable. For example, the function f(x) = |x| is continuous at all points, but is not differentiable at x = 0 because the left and right limits of the difference quotient do not agree.

Similar threads

  • Calculus and Beyond Homework Help
Replies
4
Views
607
  • Calculus and Beyond Homework Help
Replies
4
Views
756
  • Calculus and Beyond Homework Help
Replies
6
Views
914
  • Calculus and Beyond Homework Help
Replies
18
Views
1K
  • Calculus and Beyond Homework Help
Replies
5
Views
683
  • Calculus and Beyond Homework Help
Replies
5
Views
799
  • Calculus and Beyond Homework Help
Replies
2
Views
597
  • Calculus and Beyond Homework Help
Replies
3
Views
636
  • Calculus and Beyond Homework Help
Replies
10
Views
554
  • Calculus and Beyond Homework Help
Replies
6
Views
615
Back
Top